subject
Mathematics, 20.06.2021 05:30 nckishaford

If and ​, estimate with n = 13 and p = 0.5 by using the normal distribution as an approximation to the binomial​ distribution; if np5 or nq​5, then state that the normal approximation is not suitable

ansver
Answers: 1

Another question on Mathematics

question
Mathematics, 21.06.2019 19:00
D(5, 7). e(4,3), and f(8, 2) form the vertices of a triangle. what is mzdef? oa. 30° ob. 45° oc. 60° od 90°
Answers: 1
question
Mathematics, 21.06.2019 19:50
Is it greater than or less than7*3_9*2
Answers: 2
question
Mathematics, 21.06.2019 22:30
Will mark brainlist what is the slope of the line passing through the points (-2, -8) and (-3,-9)? -7/5-5/71-1
Answers: 2
question
Mathematics, 21.06.2019 23:30
Write an inequality for this sentence the quotient of a number and -5 increased by 4 is at most 8
Answers: 1
You know the right answer?
If and ​, estimate with n = 13 and p = 0.5 by using the normal distribution as an approximation to t...
Questions
question
Mathematics, 10.03.2021 19:40
question
Mathematics, 10.03.2021 19:40
question
Mathematics, 10.03.2021 19:40
question
Mathematics, 10.03.2021 19:40
question
Social Studies, 10.03.2021 19:40
question
Mathematics, 10.03.2021 19:40
Questions on the website: 13722367